Re: [考試] 熱力學一題 已解決

看板 Gov_owned
作者 gtomina8810 (雞踢偶米那)
時間 2020-10-25 07:11:00
留言 32則留言 (9推 0噓 23→)

: 各位大大好 : 我想問前年國營專業B的第三題 : 這題的題目要問壓縮機的功率,且有考慮出口速度 : https://i.imgur.com/UlFELYN.jpg
: 這是開放系統第一定律的輸入公公式 : https://i.imgur.com/hK082p5.jpg
: 但歷屆試題本的解答卻不把動能變化考慮進去 : https://i.imgur.com/ZwRVfzJ.jpg
: 不知道是我哪裡觀念有問題,再麻煩各位大大了! 怕講不清楚還是回個文好了 首先drunk大給的這個 https://i.imgur.com/Q9GGjgz.jpg
就跟描述說的一樣這個是在pump或渦輪(通常是pump) 渦輪因為都在蒸氣下運作,而且壓力很高,不太可能用不可壓縮流來計算 然後這個的用途是在當你不知道系統是走甚麼過程時 (題目沒說多變、等溫或可逆絕熱) 才能這樣用,如果不絕熱還要把Q熱量算進來 因為功本身不是狀態函數,一般不能直接積分,才要用能量的方式去回推 (這個動力學功能法那張有講) 開放系統本身多變過程(n不等於1)或等溫(n=1)功的定義為PV圖的面積 Win=積分VdP、 封閉系統也是,只是功是以輸出來定義 Wout=PdV動位能一般不會算進來 不清楚的話可以畫一下PV圖就能知道 這邊修正我推文說法,確實如果走定義反推可以得知可逆功(多變過程)有會有動位能項 所以會第一題的壓縮功會有多動能項這點沒錯 但第二題用開放系統第一定律還是要把動能項放入能量。 https://imgur.com/a/v8Luxdf 也就是動能的功會=動能的能,所以不會是6xkW 相當於只會考慮多變系統本身的功=>焓變化+散熱量 應該會是23.69kW左右 -- 這個補充一下,我其實是偷懶把之前算的Win(不含動能項)直接拿來計算 然後不管是左邊的輸入給動能用的功,還是右邊的動能會被約掉(因為同項) 功是藉由動能+PV功推出來的(實際上是外部輸入,如果是現實就是馬達或電) 那這個功帶回去算時,右邊能量的動能就不應該省略 不然等於是把這個動能偷來當輸入了吧 另外原po的第二題詳解的解答不是負的嗎?

※ 批踢踢實業坊(ptt.cc), 來自: 114.37.69.32 (臺灣)
※ 文章網址: https://www.ptt.cc/bbs/Gov_owned/M.1603581063.A.4B9.html

a88573081: 大大您好,所以大大意思是動能項除了要考慮進壓縮機做 10/25 12:23

a88573081: 功裡,也要考慮進去系統的儲能裡面嗎? 10/25 12:23

gtomina8810: 因為功是用考量動能的功能原理推出的 10/25 12:47

gtomina8810: 當然同樣的系統動能就要算進能量裡啦 10/25 12:47

gtomina8810: 就是用同一條式子導出的 帶回去算熱能是就要用同一條 10/25 12:50

gtomina8810: 我也找過還真的沒很少文章講考慮動能的(一般是忽略 10/25 13:23

gtomina8810: 所以只能從定義去回推了 10/25 13:23

gtomina8810: 但這題其實出的很違反物性 壓升速度一般會降才對 10/25 13:24

gtomina8810: 所以入口動能都不考慮了 出口怎會有動能 10/25 13:24

a88573081: 真的很想知道國營出題老師是看哪本課本出的XD不過還是 10/25 13:35

a88573081: 謝謝大大給了新的觀念! 10/25 13:35

gtomina8810: 國營出題有時候是找研究所題目改數字的 10/25 13:46

gtomina8810: 所以才會有一些算起來很詭異的東西 10/25 13:47

gtomina8810: 比較討厭是市面詳解也是會有錯 就很尷尬 10/25 13:58

gtomina8810: 好歹也是付錢買的 10/25 13:58

drunk0102: 你的意思是說 壓縮功 和最後再算Q的等式右邊都要有動能 10/25 16:45

drunk0102: 項嗎? 所以會和Win的動能互相抵銷是嗎? 因為我看你計算 10/25 16:45

drunk0102: 的Win沒有加入動能項耶 不就是原po解答的答案嗎 10/25 16:45

gtomina8810: 但第一題的功要把給動能的項加入沒錯 所以詳解有誤 10/25 16:58

gtomina8810: 我這個列式有點雜亂但最後的等號不是寫Win 是VdP喔 10/25 17:00

drunk0102: http://i.imgur.com/69vrMrr.jpg 10/25 17:16

drunk0102: 圖沒轉好= = 所以您是這個意思嗎 這個算法我之前也有 10/25 17:16

drunk0102: 想過 會不會動能最後會消掉 10/25 17:16

gtomina8810: https://imgur.com/a/JEEIFHY 10/25 17:24

gtomina8810: 對 不過我不能保證改考卷的主管有沒有其他想法就是. 10/25 17:25

gtomina8810: 上面是寫比較正式的列法 10/25 17:32

xo4n3xji4dk4: 我已經不想搞懂了哈 還是謝謝板上大神XD 10/25 19:11

drunk0102: 如果真的考這種題目... 就看筆墨分數了XD 因為也沒有一 10/25 19:44

drunk0102: 個正式的解答QQ 10/25 19:44

brunomarsfan: 推觀念 不要只背公式== 10/28 17:25

Rexspinner: 同樣認為gt大的觀念才是正確的,王立宏的詳解在算1st 11/02 20:30

Rexspinner: law時沒道理不在右邊也加入動能項 11/02 20:30

您可能感興趣